What is Electromagentic: Definition and 60 Discussions

The electromagnetic spectrum is the range of frequencies (the spectrum) of electromagnetic radiation and their respective wavelengths and photon energies.
The electromagnetic spectrum covers electromagnetic waves with frequencies ranging from below one hertz to above 1025 hertz, corresponding to wavelengths from thousands of kilometers down to a fraction of the size of an atomic nucleus. This frequency range is divided into separate bands, and the electromagnetic waves within each frequency band are called by different names; beginning at the low frequency (long wavelength) end of the spectrum these are: radio waves, microwaves, infrared, visible light, ultraviolet, X-rays, and gamma rays at the high-frequency (short wavelength) end. The electromagnetic waves in each of these bands have different characteristics, such as how they are produced, how they interact with matter, and their practical applications. The limit for long wavelengths is the size of the universe itself, while it is thought that the short wavelength limit is in the vicinity of the Planck length. Gamma rays, X-rays, and high ultraviolet are classified as ionizing radiation as their photons have enough energy to ionize atoms, causing chemical reactions.
In most of the frequency bands above, a technique called spectroscopy can be used to physically separate waves of different frequencies, producing a spectrum showing the constituent frequencies. Spectroscopy is used to study the interactions of electromagnetic waves with matter. Other technological uses are described under electromagnetic radiation.

View More On Wikipedia.org
  1. user2539

    Magnetic equivalent circuit for a UU-core inductor

    Hello, I am trying to develop a magnetic equivalent circuit for a UU-core inductor with two parallel bus bars in the middle of the slot. I am not sure as to how to model the MMF in such problem. The logical way in my mind is to have two current sources to represent the fields of each bus bar...
  2. X

    Electrons enter charged capacitor

    I cannot understand. I think that the constant electric fields and the direction the electrons ahead of cannot change the speed, but I can’t be sure about that. I really hope that someone can help me on that, thank you very mouch.
  3. R

    Engineering How to calculate the interference voltage?

    Hello everyone, I have this circuit and I need to determine the interference voltage. I am not sure, where the interference voltage is applied and how to determine it. I think the interference voltage is uv nad my equivalent circuit looks like this: Am I in heading into the right direction...
  4. Boltzman Oscillation

    I How does the magnetic component of an EM wave affect surroundings?

    Hi all, as we all know EM waves are made up of magnetic and electric waves in a plane perpendicular to the direction of propagation. Given this, why don't I see conductors being affected when I shine light at them? Woulnt the magnetism cause a force? Is is that the force is too small? What am i...
  5. Woomir

    I Why is my magnetic levitation experiment not working?

    [Mentor Note -- Two similar thread starts merged into one] Hey all, I have been working on this project for a while now, which features suspending an object midair through the use of magnetic levitation principles. Cool right? And so I have been researching and trying, even finding ground...
  6. P

    Using Lorentz Force to derive V = E/B

    Suppose the E-field is ##-E_y\hat y##, and B-field is ##B\hat z##. Mass is ##m##. z | |_____x / y ##m(\ddot x \hat x + \ddot y \hat y) = q(-E_y \hat y + (v_x \hat x + v_y \hat y) \times B \hat z)## By grouping terms with ##\hat x## and ##\hat y## together, ##m\ddot x = -qv_yB##...
  7. H

    Motion of a particle in a magnetic field

    Hi, I have to find the motion of a particles ##(x,y,z)##. However, I'm not sure where to begin. Is it correct to split the problem and first find what's the motion in the x direction then y and z. For exemple, ##m \frac{d^2x}{dt^2} = -kv_{0x} + qv_{0x}B sin 90 ## ##m\int\int...
  8. S

    Confusion about the right hand rule (Lorentz Force)

    Hello. I am trying to do a basic experiment to demonstrate I⊗β≈ƒ or current cross magnetic field results in a force perpendicular to the plane of the other two. But every time I test this out, it appears I get a force in the same direction as the magnetic field. Here is my experiment: Strong...
  9. M

    Does Moving Coils in Electromagnets Create Current? Explained

    A small coil is moved forward (without turning) between the poles of the electromagnet. Does an electric current develop in the coil? Explain the answer
  10. peace

    The motion of a charged particle in a magnetic field

    qvB=mv^2/R R=mv/qB= p/qB ! As you can see, the difference between this relation and the relation in question is in 'c'. Maybe my way is wrong. Maybe I should get help from relativity because the speed of light is involved here. Please help. Thankful
  11. J

    A Nonlinear Wave Equation (Nonlinear Helmholtz)

    I am trying to solve a PDE (which I believe can be approximated as an ODE). I have tried to solve it using 4th Order Runge-Kutta in MATLAB, but have struggled with convergence, even at an extremely high number of steps (N=100,000,000). The PDE is: \frac{\partial^2 E(z)}{\partial z^2} +...
  12. Nathan M

    Understanding frequency calculations of an object (e.g human body)

    Apologies for the noob question. The frequency of the human body can be calculated based on weins law as follows: Wavelength = 0.002898/310 = 0.00000934838 m Plugging this into the equation λν = c we get frequency = 299792458 / 0.00000934838 = 32069 Ghz Elsewhere, I see that the resonant...
  13. bhaskarporey

    About changes in a magnetic field

    Here i tried this way (see picture) Please tell me am i right or wrong. Also they says find the change in magnetic field with time using Faraday's law in a rectangular loop. How can i solve that??
  14. peace

    Elliptically polarized light & partially-polarized light

    How to distinguish elliptical polarized light from partially-polarized light?
  15. peace

    Mutual inductance between a very long, straight wire and a Semicircular loop

    I think I have to assume a point like P in the semicircle. The point in terms of r and θ: P (r,θ). So the magnetic field at that point: B = µI/2π(R+rcosθ) . So the magnetic flux: Φ = ∫B.dA= µI/2π ∫∫ rdrdθ / R+rcosθ . Is this the correct solution?
  16. V

    Time derivative jump of the electric/magnetic field

    So I just wanted to see if anyone could offer some suggestions. So in my mind this seems impossible, in the case of electric field a jump in time derivative of that field would indicated in my mind that electric charge was either introduced or removed from the system instantaneously which would...
  17. P

    Circular loop of wire is concentric with a solenoid

    Question: In Figure (a), a circular loop of wire is concentric with a solenoid and lies in a plane perpendicular to the solenoid's central axis.The loop has radius 6.13 cm. The solenoid has radius 2.07 cm, consists of 8230 turns/m, and has a current i_sol varying with time t as given in Figure...
  18. Baibhab Bose

    Fringe spacing of the Interference pattern due to two Coherent waves

    Homework Statement: In the attached image. Homework Equations: formulas of fringe width and phase differences I think. It has been a long time since I have dealt with these kinds of interference/fringewidth problem, I can't figure out a way to start solving this problem. I was thinking about...
  19. George-M

    Number of Turns vs. Wire Gauge (BLDC Motor)

    Summary: For the same mass of copper wire, would more turns or greater thickness create a stronger magnetic field from a coil? So I am attempting to make a brush less DC motor, and I am wondering whether the coils would create a greater magnetic force if there are more turns or if the wire is...
  20. QuasarBoy543298

    Exploring Gauge Symmetry in Classical Field Theory

    hi, I'm currently taking a classical field theory class (electromagnetism in the language of tensors and actions and etc) and we have just encountered the gauge symmetry, that is for the 4 vector potential we can add a gradient of some smooth function and get the same physics (if we take Aμ →...
  21. A

    Understanding Polarization-Dependent Phase Shifts of Radio Waves

    I've seen this video: There it is explained that an electromagnetic (here radio) wave has a phase shift if it was radiated in horizontal polarization, but it does not experience the phase shift when it was emitted vertically polarized when it gets reflected on the ground. When reading up on...
  22. Javier Lopez

    Particle movement in a non-static magnetic field

    Particles follows accordingly the general equation: $$ m*\overrightarrow{a}=q*\overrightarrow{E}+q*\overrightarrow{v}\wedge \overrightarrow{B} $$ But in the case of two coupled coils the time varying current at primary coil (and its magnetic field variation) creates corresponding varying...
  23. J

    Getting very low flux linkage in ANSYS Maxwell

    I'm new to ANSYS Maxwell and I'm trying to follow the inductance calculation example in the user's guide. The results for the inductance are pretty close but the value of flux are very low. What cause this? What should I do?
  24. N

    Calculate the power measured by the detector at distance h from the source

    Homework Statement An isotropic point source radiates electromagnetic energy, and its output is measured by a thin disc-like detector of radius R. Calculate the power measured by the detector at distance h from the source, assuming that the plane of the disc is orthogonal to the line of sight...
  25. E

    How do transformers work, and how to read dot notation

    <Moderator's note: Moved from a technical forum and thus no template.> Hi all, I have attempted this question but have a few queries on how transformers work, and what the dot notation represents. (a) The flux would be clockwise around the iron core. (b) This is the question where it gets a...
  26. Samuel Williams

    Are longitudinal magnetic waves possible?

    Homework Statement Are longitudinal magnetic waves possible? Give reasons for your answer. Homework Equations Working with Maxwell's equations, Lorentz force, electrostatic and electromagnetic waves in plasma. The Attempt at a Solution No idea whatsoever. I believe it is possible based on...
  27. T

    I One dimensional wave, function of a wave

    I am currently reading through 'Optics' by Eugene Hecht chp 2 page 20, he talks about the function of the wave and the direction of travel of the wave i.e ##\psi(x)=f(x-t)## and right at the bottom of the page he say this: Equation (2.5) is often expressed equivalently as some function of ##t -...
  28. B

    B Radio Waves & Penetration: Why Can They Pass Through Walls?

    Why do radio waves pass through walls while visible light can't? I guess this has something to do wit the wavelength. What I know is that higher wavelength means higher ability of penetration. So why radio waves penetrate obstacles?
  29. K

    The height of a dielectric material between two coaxial pipes

    Homework Statement This is the exercise 10.6 from Feynman lectures on Physics 2. Two coaxial pipes of radii a and b(a<b) are lowered vertically into an oil bath. If a voltage V is applied between the pipes, show that the oil rises a height H. Show that H=(V^2)(κ-1)ε_0/[ln(b/a)ρ(b^2-a^2)g] where...
  30. U

    Calculation of Change in Magnetic Flux Linkage Across a Wire

    Homework Statement A straight wire of length 0.20m moves at a steady speed of 3.0m/s at right angles to a magnetic filed of flux density 0.10T. Use Faraday's law to determine the e.m.f. induced across the ends of a wire. Homework Equations E= Nd Φ/dt but N=1 so E= dΦ/dt The Attempt at a...
  31. A

    Duration of electromagnetic disturbance

    Let us say I have a moving charge. At each point x,y,z in it's path from understanding there is a transverse electromagnetic wave being radiated (could also be viewed as a photon). The electric field at any point x1,y1,z1 in the path is disturbed. The moving charge does the same thing all...
  32. J

    Radar Cross section Vs Reflection Coefficient

    I am working on HFSS and designing Antennas for LOW RCS , in all previous works they calculate the S11 parameter for the unit cell then make an array form this unit cell and calculate the RCS , my question is what is the relation between S11 and RCS and why we calculate S11 for the unit...
  33. P

    Why is there interference if EM waves don't interact?

    Radio waves pass through everywhere without interacting with each other and that enables us to hear different phone calls and radios without disturbance. However, we do hear some noise sometimes because different signals interfere with each other. How are both of the last two statements true (if...
  34. G

    Sign of the Faraday-Lenz equation

    Homework Statement Hello, good afternoon, I have a question that's coming up when I'm doing electromagnetic induction exercises. In some exercises they ask me to determine the electromotive force (iee), and when they do the exercise resolution they do it with absolute value. In other...
  35. P

    Simulating a Faraday Cage

    Hi guys, I'm wanting to simulate faraday cage form, analysing mesh perforations defined by wavelengths of different electromagnetic spectra (eg. radio, microwave, UV etc). Are there any good analysis softwares which I could import a pre-made digital model with these perforations and simulate...
  36. G

    Spherical conductor shell problem

    Homework Statement Consider a spherical conducting shell with inner radius R2 and outer radius R3, that has other spherical conductor inside it with radius R1 (this one is solid). Initially the 2 spheres are connected by a wire. We put a positive charge Q on the sphere and after some time we...
  37. A

    Draw field lines for both magnetic fields

    Homework Statement Homework Equations N/A The Attempt at a Solution Im not sure if I've done this right because it seems too little for 6 marks. I have arrows coming from the north to the south end for the magnet's magnetic field. I have arrows going counter clockwise around the conductor...
  38. S

    What is magnetic force generated by planar spiral coil?

    Hello everybody, I have a planar spiral coil with wire. I want to move a magnet in vertical direction with the magnetic force generated by coil. I want to know how much impulsion to magnet will be occurred on vertical direction. I have been searching for the answer but I am a little bit...
  39. N

    Why a conductive shield doesn't block the magnetic field?

    Hi While studying the shielded wires, i noticed that the magnetic field of the inner conductor can penetrate the shield conductor (can be calculated in the region 3). However, the boundary condition of the magnetic field at the surface (between dielectric and perfect conductor) of a perfect...
  40. Metals

    B Photoelectric effect and atomic excitation

    A few quick questions I'd like cleared up:1) Alkali metals are said to have a really low threshold energy, enough for visible light to cause the photoelectric effect. Does this mean if I aim a flashlight (turned on) at a piece of sodium, I could ionise it? Simply flashing a light over a piece of...
  41. F

    A Experiment design FTL particle though magnetic field

    Hi. I was browsing Wikipedia on a topic, went to magnetic bearings, etc etc...ended up on synchrotron radiation then the phrase about how a moving or accelerat/ing/ed particle passing through a magnetic field generates electromagnetic radiation. Then I has a revelation. How to get a particle...
  42. D

    Solenoid Diagram Homework: Right Hand Rule Explained

    Homework Statement (http://imagemap-generator.dariodomi.de/uploads/160725_010743_e42ea59f2569dfbV.png [Broken]) Homework Equations No equations or anything. Just need a visual concept understanding (11U Physics) The Attempt at a Solution The solutions say that all compasses should point...
  43. alxkrgr

    Where to place a third charge so it experiences no force

    Homework Statement A point charge of 5.6 microcoulombs is placed at the origin (x=0) of a coordinate system, and another charge of -1.9 microcouloumbs is placed on the x-axis at .29m. Where on the axis can a third charge be placed so that it experiences no charge? q1= 5.6 E-6 q2= -1.9 E-6 d=...
  44. Surya97

    I Electric and Magnetic Fields, Photons, Charges

    I know that magnetic fields create moving charges (an electric current) normal to the plane of the magnetic force lines. I also have heard that magnetic and electric fields create each other in a perpendicular direction to the other (badly worded). Electric currents are moving charges (usually...
  45. Dong Hoon Lee

    How to Convert Vectors to Spherical Coordinates at Given Points?

    Homework Statement transform the following vectors to spherical coordinates at the points given 10ax at P (x = -3 , y = 2, z=4) Homework Equations x y z can be chage into x = rsinθcosφ , y=rsinθsinφ , z=cosθ The Attempt at a Solution ax vector can be expressed ar,aθ,aφ so, I can change x ...
  46. Dong Hoon Lee

    Transform Vectors to Spherical Coordinates at P (-3,2,4)

    The problem is << transform the following vectors to spherical coordinates at the points given 10ax at P (x = -3 , y = 2, z=4)>> Actually, My first language isn't English, please understand that. x y z can be chage into x = rsinθcosφ , y=rsinθsinφ , z=cosθ ax vector can be expressed...
  47. Z

    Calculating voltmeter resistance in DC circuit

    Homework Statement initial problem: A 228 −Ω resistor and a 586 −Ω resistor are connected in series across a 90.0−V line. A voltmeter connected across the 228 −Ω resistor reads 24.0 V . Find the voltmeter resistance. For Req, i got Req=228R/(2+228), R being the resistance of the voltmeter. But...
  48. J

    Index of Refraction: Calculate ni for 10 cm Attenuation

    Homework Statement Within a certain material, an EM wave with = 1 mm is attenuated to 10% of its original intensity after propagating 10 cm. Determine the imaginary part of the index of refraction ni Homework Equations 3. The Attempt at a Solution [/B] so...
Back
Top